Are the Row Vectors of a Matrix with Orthonormal Columns Also Orthonormal?

  • Thread starter Thread starter mathboy20
  • Start date Start date
  • Tags Tags
    Row Vectors
Click For Summary
The discussion centers on whether the row vectors of a matrix with orthonormal columns are also orthonormal. The matrix A is defined as having columns that are the standard basis vectors in R^n, which are orthonormal by definition. The user seeks to demonstrate that the rows of A are also orthonormal, suggesting that since A^(-1) = A^T, the orthonormality of columns implies the same for rows. Another participant points out that if the columns are standard basis vectors, the rows will be the same, thus trivially orthonormal. The conversation hints at a broader proof for any set of orthonormal vectors, not just the standard basis.
mathboy20
Messages
27
Reaction score
0
Hi

Given A an n x n matrix where the columns are the set of standard vectors S = {e1, e2,...e_n} in R^n

These vectors are orthonormal according to the definition since

<e_i, e_j> = 0, i \neq j

<e_i,e_i> = 1

Since the columns of A then they are orthonormal according to the definition, and therefore the matrix A is orthogonal matrix which implies that

A^T A = I <---> A^ (-1) = A^T, and so on.


I need to show that the row vectors of of A are orthonormal too.

Any idears on how?

My own idear is that

Since

A^ (-1) = A^T, then the columns in invertible A become the rows in transposed A, and the these orthonormal according to the definition?

Sincerely Yours
Mathboy20
 
Physics news on Phys.org
I don't know what your reasoning is, but with A^(-1) = A^T then you also know that (A^T)^(-1) = A. A^T*A = A*A^T. It follows from there.
 
Actually, the way you stated the problem, its trivial. If the columns of the matrix are just the standard basis vectors e1, etc. then the rows are those basis vectors too! More likely you want to prove that if columns are any set of orthonormal vectors, then the rows are too. Orthodontist's hint is good.
 
Question: A clock's minute hand has length 4 and its hour hand has length 3. What is the distance between the tips at the moment when it is increasing most rapidly?(Putnam Exam Question) Answer: Making assumption that both the hands moves at constant angular velocities, the answer is ## \sqrt{7} .## But don't you think this assumption is somewhat doubtful and wrong?

Similar threads

  • · Replies 8 ·
Replies
8
Views
1K
  • · Replies 3 ·
Replies
3
Views
2K
  • · Replies 7 ·
Replies
7
Views
2K
  • · Replies 2 ·
Replies
2
Views
2K
  • · Replies 14 ·
Replies
14
Views
4K
  • · Replies 2 ·
Replies
2
Views
2K
  • · Replies 5 ·
Replies
5
Views
2K
  • · Replies 4 ·
Replies
4
Views
2K
  • · Replies 11 ·
Replies
11
Views
3K
  • · Replies 10 ·
Replies
10
Views
2K